17
$\begingroup$

In an ordinary category $C$, one says that an object $X$ is $\kappa$-compact if the representable functor $Hom(X,-)\colon C \to Set$ preserves $\kappa$-filtered colimits. We say $C$ is locally presentable if it is cocomplete and "generated" by $\kappa$-compact objects for some $\kappa$.

In an $(\infty,1)$-category $C$, one says that an object $X$ is $\kappa$-compact if the representable functor $Hom(X,-)\colon C \to \infty Gpd$ preserves $\kappa$-filtered $(\infty,1)$-colimits. We say $C$ is locally presentable if it is cocomplete and "generated" by $\kappa$-compact objects for some $\kappa$.

There are many equivalent, also analogous, definitions in both cases.

An $(\infty,1)$-category is locally presentable if and only if it admits a presentation by some locally presentable, cofibrantly generated model category. However, the only proof of this fact that I have seen (in A.3.7.6 in Higher Topos Theory) uses a different equivalent definition of both notions (as an accessible localization of some presheaf category). Thus my question:

Is there any relationship between an object $X$ being $\kappa$-compact in a locally presentable model category and being $\kappa$-compact in the $(\infty,1)$-category that it presents?

$\endgroup$

1 Answer 1

20
$\begingroup$

If $\mathcal{C}$ is a combinatorial model category, then for all sufficiently large regular cardinals $\kappa$, an object of the underlying $\infty$-category is $\kappa$-compact if and only if it can be represented by a $\kappa$-compact object of $\mathcal{C}$. The meaning of "sufficiently large" might depend on $\mathcal{C}$.

If $\kappa$ is not sufficiently large, then you generally don't have such an implication in either direction. For example, every finitely presented group is a compact object when viewed a (discrete) simplicial group, but need not be a compact object in the associated $\infty$-category (this requires that the classifying space of the group be finitely dominated). On the other hand, any space which is finitely dominated (i.e., a homotopy retract of a finite cell complex) is a compact object in the $\infty$-category of spaces, but cannot be represented by finite simplicial set unless its Wall finiteness obstruction vanishes.

(These counterexamples are not particularly compelling: you could get around the first one by restricting your attention to cofibrant objects, and the second one is very particular to the cardinal $\omega$. So perhaps there is something better to say.)

$\endgroup$
6
  • $\begingroup$ Thanks! I'm happy to assume that $\kappa$ is sufficiently large. Where can I find a proof of your first statement? $\endgroup$ Apr 25, 2012 at 21:06
  • $\begingroup$ Also, my next question is about an analogous statement for relatively $\kappa$-compact morphisms. I hope it would be possible to deduce that from the absolute version, but maybe you know a reference for the relative case too? $\endgroup$ Apr 25, 2012 at 21:14
  • $\begingroup$ One direction is fairly straightforward: if $\mathcal{C}$ is combinatorial, then there exists a cardinal $\kappa$ such that weak equivalences in $\mathcal{C}$ are closed under $\kappa$-filtered colimits. Then $\kappa$-filtered colimits are also homotopy colimits, so the functor $F$ from $\mathcal{C}$ to its underlying $\infty$-category preserves $\kappa$-filtered colimits, and in particular is accessible. It then follows that $F$ preserves $\tau$-compact objects for $\tau$ sufficiently large. $\endgroup$ Apr 26, 2012 at 2:55
  • $\begingroup$ The reverse implication seems a little trickier. One way to prove it is to first argue that it depends only on the Quillen equivalence class of $\mathcal{C}$ (in other words, only on the underlying $\infty$-category of $\mathcal{C}$). This lets you assume that $\mathcal{C}$ is a Bousfield localization of simplicial presheaves on some small simplicial category. From here it's not hard to reduce to the case where $\mathcal{C}$ is the category of simplicial sets, in which case the result is true for any uncountable $\kappa$. $\endgroup$ Apr 26, 2012 at 3:02
  • $\begingroup$ I'm afraid I don't know references. $\endgroup$ Apr 26, 2012 at 3:03

Your Answer

By clicking “Post Your Answer”, you agree to our terms of service and acknowledge you have read our privacy policy.

Not the answer you're looking for? Browse other questions tagged or ask your own question.